Math, asked by charvikreddy8100, 3 months ago

The third number is the HCF of first two numbers What are the three numbers ?
A. 8, 60, 14
B. 40, 56, 8
C. 4, 20,5
D. 2, 3, 4​

Answers

Answered by MaheswariS
2

\textbf{Given:}

\textsf{Third number is the HCF of  first two numbers}

\textbf{To find:}

\textsf{The correct option from the following:}

\mathsf{A.\;8,60,14}

\mathsf{B.\;40,56,8}

\mathsf{C.\;4,20,5}

\mathsf{D.\;2,3,4}

\textbf{Solution:}

\textsf{Consider, the numbers 8 and 60}

\mathsf{8=2^3}

\mathsf{60=2^2{\times}3{\times}5}

\mathsf{Since\;2^2\;is\;common,\;HCF=4}

\textsf{Consider, the numbers 40 and 56}

\mathsf{40=2^3{\times}5}

\mathsf{56=2^3{\times}3{\times}7}

\mathsf{Since\;2^3\;is\;common,\;HCF=8}

\implies\mathsf{HCF\;of\;40\;and\;56\;is\;8}

\implies\boxed{\mathsf{Correct\;option\;is\;B}}

\textbf{Find more:}

Find the ratio between HCF and LCM of 10 15 and 20

https://brainly.in/question/15064394

Find the greatest number that divides 3128 and 420 ​

https://brainly.in/question/15327150

Find hcf of 144, 96, 54 by prime factorisation

https://brainly.in/question/2132704

Find the HCF of 252525 and 363636​

https://brainly.in/question/11206176

Answered by pulakmath007
5

SOLUTION

TO CHOOSE THE CORRECT OPTION

The third number is the HCF of first two numbers What are the three numbers

A. 8, 60, 14

B. 40, 56, 8

C. 4, 20,5

D. 2, 3, 4

CONCEPT TO BE IMPLEMENTED

1. HCF = Highest Common Factor

2. For a given set of numbers their HCF must divide each of the numbers

EVALUATION

CHECKING FOR OPTION : A

14 does not divide 8 and 60

So 14 cannot be HCF of 8 and 60

So this option is not correct

CHECKING FOR OPTION : B

8 does not divide 40 and 56

So 14 can be HCF of 40 and 56

More precisely

40 = 2 × 2 × 2 × 5

56 = 2 × 2 × 2 × 7

HCF = 2 × 2 × 2 = 8

So HCF of 40 and 56 = 8

So this option is correct

CHECKING FOR OPTION : C

4 does not divide 5

So 4 cannot be HCF of 20 and 5

So this option is not correct

CHECKING FOR OPTION : D

4 does not divide 2 and 3

So 4 cannot be HCF of 2 and 3

So this option is not correct

FINAL ANSWER

Hence the correct option is B. 40, 56, 8

━━━━━━━━━━━━━━━━

Learn more from Brainly :-

1. If HCF of two numbers be 40 then which of the following cannot be their LCM.

https://brainly.in/question/28609013

2. The HCF and LCM of two numbers are 17 & 1666 respectively. if one of the numbers is 119 find the other

https://brainly.in/question/13812250

Similar questions